You are on page 1of 128
Footman amentcad Contents History taking format 4 1. Dizziness (Cereballar metastases secondary to lung carcinoma) ...n.nnnmnnnnnnssnnnnnnnee Infertility Headache examination (Temporal arteritis). GERD (pediatric)... 12 Intussusception. 15 is, Allergic rhinitis (hay fever). 2, 3 4, 5, 6. Hepat 7, 8 Ear examination es ” a 9. Cholesteatoma.. 10. Testicular torsion... 27 11, MMSE. a BB 1 RR nn 28 12, Threaten abortion with hyperemesis gravidarum... 31 13, GBS +ve in pregnancy . 32 14. Advanced age pregnancy. 33 45. Pre pregnancy DM counselling. 36 16. Thyroid examination .esnnnn penn : 40 17. Threaten abortion with retroplacental blood clots. 18. _ Retained placenta with shock. 19. Hip Examination. Hip osteoarthritis. Adductor tendonitis.. Trochanteric bursit 20. Low lying placenta ... 21. Post-natal checkup_gestation DM pt nnn ne 51 22. Pelvic examination PE... 23. immunization counselling... 24. Osteoporosis. 25. Implantation bleeding. 26. Primary hyperparathyroidism... 27. POST PARTUM CHECK UP (Post partum PE post 6wks). 69 28, Secondary metastasis. 29. Contraceptive counselling . Footman amentcad 30. Bedwetting... 7 31, Mental Health Act. 78 32. Scrotal swelling PE 80 33. Duodenal atresia 84 34, Mitral stenosis. 85 35. Oesophageal stricture... 87 36. Seizure. 89 37. Throat pain PE. 38. HRT counselling... 39. Syncope due to prolonged QT interval. 40. Giadiasis... 41. Headache..... 101 102 42. Upper limb examination 43, SCC. 44, Hypocalcemia (hypoparthyroidism) PE. 107 45. GAD. 109 46. Chlamydia 110 47. Hand examination PE. a 48, Toddler diarrhoea... 1d 49. Lichen sclerosis... 116 50. Ectasy withdrawal + diazepam + suicidal attempt. senntnnnnnenmnrinee 11 BL UT nnn 124 52. Anterior cruciate ligament taf scan ennnessninnnnnnnnnnnnniinnnninnnnnssnnnnee 1B History taking format Se ae ‘rng ‘awe perience eer tule omen ‘in odors ann? ‘Seep soos coat oe ose ersten Sie an yr Sse ne rn uF wd et NE ans ert eng an Yonge ‘vse hors Roses fogeame om ae sonmy Eee arent eamennones SEE as a __ eee emins annette rte et ‘ce EStan tases haces aa Sisters dey tom ni ‘tos tps er Tn Os pay ate Ger a eS ee ie cos nh {Crefecteroe tomes eons emoren fre ocd pes apron nor son Paes M00 ron, eae pds etn ocr meter heb hee eset ‘pet mesmeactnag ‘sonra eat ‘Seen en ‘ene eee ‘aurascn outs den) Soman pak ae Sa Eh pan nt SET ce an te Footman amentcad 1. Dizziness (Cereballar metastases secondary to lung carcinoma} You are a junior doctor working in the rural emergency department. Your next patient is Mr. Jones, a 89 year old retired accountant, who presents with increasing dizziness for a couple of months which severely reduces his daily activities. YOUR TASK IS TO: * take relevant history * Examine the patient + Explain the possible diagnosis and the investigation required. Positive findings History + increasing dizziness for a couple of months (around 3 months) + now it affects lifestyle (not able to do routine activities, very scared to move around due to fear of fall). + Dizziness means the patient feels the world spinning around + It is continuous and does not vary with the change in posture. + Throughout the day + No loss of conscious + Hewas fine before that. + No recent fever or ear infection, no palpitation or transient loss of consciousness + No vomiting + No restriction of Neck movement + Infection - nil + Injury trauma ~ nil + DM=nil + Hypertension — nil + Fever~nil + Mr. Jones uses a hearing aid (10 years) and does not have any problem with it + Theres no tinnitus or headache, On added sound on hearing. + He has developed increasing cough and chest pain while breathing over 3-4 months and he lost about 6-7 kgs. + No visual symtoms, nausea or vomiting + Px. + FHx: unremarkable + SHx.: single and happy man except worrying about the current problem, no medication, NKA, + smokes 20 cigarettes/day (over 40 years!!!) little alcohol. + Recreational Drug — nil + Taking care of self, perfectly healthy + CvSh/o + Resp h/o + CNS h/o Positive findings on EXAMINATION: Footman amentcad General appearance: + he looks thin or cachectic, + P72/min, regular, BP -130/80, + RR 28/min, Sa02.94 % on RA. Neurological: + Conscious, oriented, + all cranial nerves intact especially 7 and 8 (vestibulocochlear), + vision normal, + power, tone and reflexes normal. + Cerebellar signs: nystagmus, dysdiadokokinesia, rhomberg’s: -ve finger nose test positive, ataxia to the left side. Positional test or Hallpike’s manoeuvre -ve Weber’s and Rinne’s negative. cvs + no arrhythmia or carotid bruit. Respiratory system: + decreased air entry in right upper lobe, no added sounds. Performance guideline History -how are you doing now? any dizziness or drowsiness? ~can you tell me more? has this happened before? How often? do you remember if you were febrile at that time -did you get any warnings like feeling nauseous or strange smell? -any BOV, headache, fever? -any weakness, numbness or tingling? -any palpitations, chest pain or SOB? -do you black out or feel dizzy when you stand up quickly? -how’s your diet? -any trauma to the head by any chance? -did you injured yourself? -nausea and vomiting, weakness, headache? 5-General -PMH (DM, HTN, epilepsy, heart D) -medications Cough/SOB/Fever Early morning headache, woke up from sleep LOA, LOW -(key q)SADMA? (+ve alcohol wine 2-4 drinks a day) -any binge drinking? ‘Smoking 40 years -family history of sudden death, epilepsy? Footman amentcad Diagnosis : Cereballar metastases secondary to lung carcinoma. To rule out acoustic neuroma. DDx: Other differential diagnosis of Vertigo not related to this case 1) Drug induced -hypotensive drugs most common 2) Acute labyrinthitis -more common in young and middle-aged people, single episode with tinnitus and deafness, diagnosis of exclusion 3) BPPV -more common in elderly females, not accompanied by any other symptoms, positive hallpike’s manoeuvre or head position testing. 4) Meniere's disease age 30-50 years, abrupt onset, last for longer time, associated with progressive deafness, vomiting, sweating and pallor, impaired caloric test and audiometry 5) Wax in the ear 6) cerebrovascular disease ~vertebrobasilar insufficiency or multiple sclerosis ~associated typical features, 7) trauma , heart problems, infection but less likely 8)electrolyte imbalance, or very less likely epilepsy Investigation: haemoglobin, blood glucose. ECG and holtermonitor, Audiometry and caloric test. Chest x-ray (? Bronchial CA) CTor MRI Audiometry, brain evoked potentials (MS), electrocochleography, electrooculography and rotational test depending on the cause. 2. Infertility 27-year-old Janet is your next patient at your GP. She tells you that she had been trying to fall pregnant since more than 1 year now but she could not and she is quite worried about this. TASKS ‘LAppropriate history 2.Examination findings from examiner 3.Relevant investigations 4,.Management Positive findings Period is regular. No problems with periods (4-6 days and 30 days cycle) No pain or bleeding between period Pain throughout period (endometriosis) been together with partner for 2 years now no contraception partner hasn’t done evaluation for subfertility so far 7 Footman amentcad no children before this relationship either, no miscarriage, no STI no fever discharge back pain (PID) no lump on tummy (fibroid) no medication no family history of same issue no sx or procedure no DM or medical prob ‘occupation - office work. None of stress at home or work eating problem — none / excessive exercise + recent wt gain or loss, excessive hair on face and body, weather preference, discharge from nipple - none Physical Exam findings from the examiner 1-General appearance - fine -what is the BMI? 23 -Any Acne, excessive hair growth? -Any Pallor, lymph node enlargements, edema? None 2-Vital signs -What is the BP and temperature? 110/70, 36.8 3-Quick respiratory and CVS examination - normal 4-Thyroid examination - normal 5-Abdominal examination: normal 6-Pelvic examination: normal 7-Office tests Urine dipstick - normal blood sugar level - normal Performance guideline Differential diagnosis: -Unknown Infrequent sexual intercourse and ignorance about fertile period -PCOS “PID -Endometriosis “Fibroids -Thyroid disorders -hyper/hypothyroid -Hyperprolactinemia -Eating disorders Stress, exercise -Medical-DM, SLE, renal disorders -Surgical-Asherman’ssyndrome -Medications-antipsychotics, spironolactone, chemotherapeutic agents 8 Footman amentcad “SAD -marijuana -Family history of subfertility History ‘L-approaching+ confidentiality -Hi Janet, I'm Dr.___ your GP today. | can see from the notes that you have been trying to. become pregnant for 1 year now but you couldn't. | just need to ask a few questions and some of them would be sensitive and personal but let me assure you that everything we gonna talk about will be private and confidential. Would that be alright with you? 2-5 P’s questions Period -when was your last menstrual period? ~Are they regular? -How many days of bleeding and how many days apart? -Have you had any heavy bleeding or pain during menstruation? Partner (sexual): How long have you been in the stable relationship? -Has your partner done evaluation for subfertility so far? -Does your partner have any medical or surgical conditions? -Any medications that he is taking? “Sorry to ask you this, but does your partner have any children from previous relationships? -have you or your partner ever been diagnosed with sexually transmitted illnesses? Pregnancy: have you ever been pregnant before? -any previous miscarriages? Pill: what type of contraception you were on before planning for pregnancy? Pap or HPV screen: when was your last pap or hpv screening? 3-Differential diagnosis questions: infrequent sexual intercourse -how frequent do you undergo intercourse? Do you live with your partner always? (About once or twice a month or so cause we're not living together always cause he's working in something and comedown once a month) Unawareness about fertile period -Do you know about your fertile period? Fertile period is the time during which you undergo intercourse, there's a high chance of PEF (PID, Endometriosis, fibroids) -do you have any abnormal, offensive vaginal discharge? Any fever, back pain? (PID) -Any pain in between periods or during intercourse? (Endometriosis) -Any mass, lumps or heaviness that you can feel in your tummy? (Fibroids) -PCOS: Any recent weight gain, acne, excessive hair growth? -Thyroid disorders: Do you have any weather preference? How are your bowel habits? -Hyperprolactinemia: Any headache, blurring of vision, milky discharge from nipples? -Eating disorders/ stress/ Excessive exercise: Do you think that you are excessively overweight? Do you try to reduce your weight through crash dieting or excessive exercise? (Eating disorders) 9 Footman amentcad -What is your occupation? Do you have any stress at home or at work? (Stress) -How much exercise do you do? 4-General questions past surgical and medical history -Have you had any surgical procedures especially those done down below? (Asherman's) -Any medical illnesses like diabetes or renal disorders? ‘Smoking/ alcohol/ Drugs or Medications -Do you smoke, drink alcohol, or take any recreational drugs? -Do you take any Medications or over the counter medications? Family History Do you have a family history of subfertility? Thank you for those information, Janet. I'd just talk to my examiner and will get back to you shortly. Physical Exam findings from the examiner 1General appearance -what is the BMI? 23 -Any Acne, excessive hair growth? -Any Pallor, lymph node enlargements, edema? None 2Vital signs -What is the BP and temperature? 110/70, 36.8 3-Quick respiratory and CVS examination 4-Thyroid examination 5-Abdominal examination: Inspection -Any visible distention? Any mass? Palpation -Palpable mass or tenderness? (No visible distention, no mass or tenderness) 6-Pelvic examination: Consent & chaperon Inspection of vulva and vagina any bleed, discharge, vesicle, rash? (No bleeding or discharge) “Speculum exam is cervix healthy? Discharge and bleed? (No discharge or bleed from the cervix) -Per vaginal exam (CMT+ Bimanual) is there CMT? (None } What is the size, position and mobility of uterus? Any Tenderness? (Uterus size is normal, anteverted, mobile, no tenderness) -Any Adnexal mass or tenderness? (No adnexal mass or tenderness) 7-Office tests Urine dipstick blood sugar level Investigation 10 Footman amentcad -I would also like to arrange for some investigations such as FBE, U&E, LFT, TFTs, serum prolactin -urine MCS (microscopy culture and sensitivity) -Mid luteal hormone assessment (21st day serum progesterone; if >3nanograms/mL it means lady is ovulating) (Key issue) -FSH, LH, estrogen, progesterone transvaginal ultrasound (Key issue). -if everything turns out to be normal, we need to arrange for a hysterosalphingogram. It is done to check the patency of the tubes. A dye will be injected through the cervix, and we will look if the dye is passing through the tubes or not. (Note say this only if everything normal) Diagnosis and Management -As far as the details you have given me and after examining you, | could not find any organic reasons, the most likely cause is infrequent sexual intercourse and your ignorance regarding the fertile period. So I'll advise you further on this. -if you are planning for a pregnancy, it would be better to undergo intercourse 2-3 times/week. -To find out about your fertile period or whether you're ovulating or not: you can get ovulation kits from the pharmacy which are urine kits and just like a pregnancy kit. You can start testing from the eleventh day of your periods. Other methods by which you can detect your ovulation is by basal body temperature method. For this you need to get a basal body thermometer from the pharmacy, you will also get a chart along this, and you need to record the temperature from your armpit, every morning before you get out of bed, and record it on the chart. A rise of 0.2€ for 3 days over the previous 6 days temperature will tell you that you're ovulating. Next is cervical mucus method, You need to observe the cervical mucus everyday so at time of ovulation the secretion will become thin, more in amount and lubricative. After ovulation it changes to thick mucus It is always better to treat both partners so when you come for your next appointment | would like to see your partner as well | will also give you reading materials about subfertility if none then I can refer you to infertility clinic. review when results of Ix also come 3. Headache examination (Temporal arteritis) 50-year-old man, complains of right sided headache. and also pain while chewing food. bp is stable. cant remember what else was there Task 1 relevant head and neck exam 2. tell your differentials with reasons to the patient Examination steps ‘ask for pain, painkillers after allergy 2-check vital signs (pulse, BP, temp) 11 Footman amentcad 3-Look -face: asymmetry, bruises, laceration, rash -eyes: ptosis, redness, discharge “nose: runny nose -ear: discharge -mouth: dental caries, enlarged tonsils, red pharynx -temporal area: temporal artery (prominent, beaded, site) 4-Feel for tenderness -Frontal bone -temporal bone (+ve tenderness} -temporal artery (tenderness, thickness, pulseless) -scalp (+ve tenderness) -2ygoma -maxilla-> mandible -temporomandibular junction (TMi) (tenderness and crepitation) uses (maxillary, frontal, ethmoid) -teeth (tenderness on percussion) -Neck (spine and Para spinal muscle) 5-Move -neck movement + neck stiffness 6-Cranial nerve (2,3,4,6) -all especially eye movements and fundoscopy T-otoscopy!! 8-shoulder and hip movement for polymyalgia rheumatica myopathy (active and passive movements) 9-complete with Upper and lower limbs neuro, other cranial nerves, respiratory and CVS iagnosis and differentials ‘-temporal arteritis (most likely): is a condition in which the temporal arteries, which supply blood to the head and brain, become inflamed or damaged. The cause of the condition is unknown; it may be linked to the body’s autoimmune response. Usually our immune system produce antibodies to fight infections but in temporal arteritis they are fighting our own body tissues in your case the temporal artery. Causing your symptoms, 2-tension headache, migraine, cluster 3-sinusitis 4-TMI dysfunction S-cervical spondylosis 6-meningitis T-space occupying lesion 4. GERD (pediatric) Case 3 Mother of 4month old baby, comes that baby is having vomiting Tasks R Footman amentcad 1. History 2. Physical examination findings from examiner 3. Counsel mother ~ Vomiting off and on for 2 months, since birth ~ Just milk, not forcefully - Bottle feeding - Fever rash — nil - Lump and bump — nil = Low=nil - Loose motion ~ nil - Water work ok - Follow the position my baby is convenient, mostly lying - No associated problem - Wtgain and immunization up to date PE - Normal Performance guideline History Reflux Questions (= Vomiting) -Ican see from the notes that your baby is throwing up milk all the time? Since when did it start? How often? -is it getting worse? -what is the color? -what is the amount? it related to feeding? related to change in position? -anything make it better or worse? 2-associated symptoms -how is his bowel motions? -is he crying a lot while feeding? Does he draw up his legs while crying? -any fever, rash or recent viral infections? -any cough? Any difficulty in breathing? 3-Dehydration questions -does he look drowsy or irritable? -any change in the number of wet nappies? Does he cry on passing urine? Any smelly urine? -do you breastfeed or bottle feed him? Any problems with feeding? Have you introduced solid food? 4-BINDS 2B Footman amentcad -any complications during the birth? his immunization up to date? he thriving and growing normally? (will give you growth chart 50% percentile) -any siblings with similar symptoms? -do you have good support? -anyone smoke at home? Physical examination from examiner 1-General appearance (DR PJL) 2/5 3-growth chart (no need if mother gives you) 4-ENT + neck stiffness. 5-Full systemic focus on abdomen “Inspection (distension, masses) “palpation (tenderness, masses, RIF emptiness, rigidity) *auscultation (bowel sounds) *Hernia orifices genitalia and anal inspection Goffice tests Explanation condition -most likely he has Gastroesophageal reflux which is when the content of the stomach are brought back up to the food pipe or even into the mouth after each feed in otherwise healthy and thriving well baby. Cause in babies it is common for the valve at the top of the stomach to be quite loose that is why the reflux happens. Outcome let me assure you that as your baby grows older this valve will become stronger. It gradually improves with time especially after introduction of solids. It is a common condition, and natural process will resolve by itself with time. Management advice -frequent smaller feeds (not less than 3 hourly) -feed him in upright position -burb him after each feed -do not put him immediately to sleep after each feed -put him in cot with head elevated 10-20 degree Red flags vomit forcefully, not thriving, not feeding well, fever Review reading materials 14 Footman amentcad 5. Intussusception Father brings 9 months old in ED with sudden onset of crying since last 4 hours Task * History © PEFE * Dxand Mx Positive findings * Vital stable ‘Sleeping and crying and screaming for hours, never happened before, stopped after 15 min * Come and go and when comes, he screamed ‘Drawing leg to chest and clenching of fist © 3 Episodes so far © Turn blue ‘+ Not out of breath © Vomit twice, green contained food, large amount * No fever * Relating to food — not sure since not eating for last 6 hours, vomit since last night * Loose motion one time * No tummy distension, no lump in tummy ‘© Water work—no smell, color change, not crying while passing water * Since this morning, no eating at all ‘Not sleeping and crying last night + Babyisirritable Normal birth h/o, no wt loss, immu up to date * No travel, no contact with similar problem or feverish person Family h/o—nil PE * GA irritable, no pallor, moderate dehydration Vital sign —no sig © Growth chart - normal Abd —no distension, bowel sound +, sauce sage shape mass, empty signs © Hernia orifices 15 Footman amentcad © Resp—nad © UDT—normal «Peri anal inspection ~ red currant jelly stool + Dx © Intersussception © Admit © Nil per mouth * NG insertion « Wline and fluid * Blood and urine for inv * Inform to pediatric registera ‘© Will mx as MDT including pediatric, radiologist and nurse and staff + Airenema first «If fail may need surgery 6. Hepatitis Case 4 25-year-old male coming to GP clinic with temperature 38.8C, and has nausea for two days. He had been to Thailand with his girlfriend for 2 weeks. He returned 1 month ago. TASKS 1. Relevant history 2.PE findings from examiner 3.Investigations 4.Management = Not feeling good ~ Fever, nausea - Fever ~2 days, on and off, Panadol and help, no rash, no headache, no cough and sore throat, no tummy pain, - Water work—ok, no burning pain, high color for 2 days = Poo didn’t notice color change - Skin color + - Sexual —no multiple partners, stable partner for 1 yr, no STI - Street food in Thailand yes = Tattoo, piercing in body - no - Nolv 16 Footman amentcad - Smoking - 1-2 per day, off and on - Alco- occasionally Physical Examination = GA=normal - Mild dehydration - Nol = BMI=normal - Noneck stiffness = No mass on abd exam - Tenderness on rt hypochondrium - UDT + for bilirubin = Other sys - normal Performance guideline Can you tell me a bit more about it? Patient is not been eating well for the past 2 days. Fever questions: When did it start? Is it continuous or on and off? Have you taken any medication? How about your water works? Any other associated symptoms like vomiting, skin discoloration? How are your bowel motions? How about the color of your bowel? Have you noticed any pale stool or dark urine? | will be asking some sensitive questions, rest assured that everything you tell me will be kept confidential unless there is harm to you or to others, ‘Are you in a stable relationship? For how long have you been in this relationship? Any previous partners? While in Thailand, did you have unprotected sex with your girlfriend or anybody else? Did you have any streetside food or water? Where were you living? Any possibility of eating or drinking contaminated water or food? What type of water did you use to brush your teeth? Did you do any bush walking or trekking? ‘Any exposure to mosquito bites? (r/o malaria) Did you have any rash anywhere in your body? (r/o Dengue fever) Did you have any tattoos, piercing, or acupuncture? ‘Any recent blood transfusions or donations? Have you been in contact with someone with hepatitis? Have you received any vaccinations in the recent past? PHYSICAL EXAM GA: signs of liver disease -parotid gland enlargement, gynecomastia, ascites, spider erythema Vital signs Skin -any tattooing, body piercing, IV marks, scratch marks Abdomen -any distention, organomegaly, tenderness, rigi ity, guarding, ascites, bowel sounds v7 Footman amentcad Genital -testicular atrophy, lymphadenopathy Office test: UDT INVESTIGATIONS Full blood exam cRP UEC LFTs (total bilirubin, AST, ALT, ALP will be high) Hepatitis serology Peripheral blood smear (thick and thin film) MANAGEMENT You most likely have a viral infection of the liver called hepatitis, ‘As | have excluded exchange of body fluids, blood contacts, and STIs, most probably you are having hepatitis A. Itis avery common condition, and that is why you are having fever and nausea. It spreads from person to person through close contacts with use of towels, contaminated hands, contaminated food and water. You are most infective to others 2 weeks before and 1 week after yellowish discoloration of the skin Please take adequate bed rest at home. Have a fat-free diet and increase your fluid intake. Please avoid alcohol, smoking, and liver damaging drugs like paracetamol or panadol. Please avoid sharing cutlery and crockery during meals. It is important during the illness that you do not handle food. Take high carbohydrate and low fat diet. ‘The prognosis is very good and complete recovery is expected. It is a notifiable condition, we must report this to the department of health services. Am I risky to my girlfriend? For your girlfriend and other close contacts, we can give immunoglobulins, and they should be given within 2 weeks of contact. How can we prevent it next time? 2 doses of hepatitis A vaccine, given 6 months apart. (vaccine only given to those who hasn't got the infection or never had vaccine) -vaccine is protective for 20 years. 7. Allergic rhinitis (hay fever) Case S You are a GP. You are going to see a 21 years old Thomas who came to you because he is unwell. Tasks: History DDx Mx 18 Footman amentcad Positive findings = Sneezing alot - Block nose - Fever + - NoNV - Forlast few days - Sore throat, Cough — nil = Running nose +, watery on both sides, every year happen similar but this time worse, took - decongestant spray overuse - Eczema—nil = Family history — nil = SOB~nil - Bladder and bower — nil = Any regular medicine — nil = Alco ~ social drinking - Smoking = nil - No recreational drug — inhal - Early morning headache — nil = son has URTI Performance guideline History Details of runny/blocked nose -How long? Is this the first time? Is it seasonal like happening this time very year? Do you use any medications for it? How does the discharge look like? Clear or any color? Any offensive smell? Is it affecting one or both nostrils? DDx—Any watery eyes? Any sneezing? (hay fever) Any cough? Any fever? Any SOB? Any joint pain ‘or muscle pain (URTI), any medication used for blocked nose, Any headache? Bad breath? Mucus dripping down the throat? Pain on the face? (sinusitis) Any allergy or asthma history before? What about your parents? Is it triggered by any allergens?(key q) Do you have carpets or pets at home? ‘Anyone in the family having the same condition? (key q) Past medical, past surgery SADMA DDx “It’s seems like you are having a condition called allergic rhinitis or hay fever which is an allergic. response to specific allergens. It causes the inflammation of the inner lining of the nose causing sneezing and nasal discharge. Allergens are usually pollen, mold, dust or animal dander. Most common allergen is pollen. It is common in hot, dry season as there are more pollens in the air. That's why your symptoms are seasonal. 19 Footman amentcad Other possibilities are sinusitis which is the infection in the linings of the small spaces inside your nose and head, Asthma, Infection in the airways (URTI) or very less likely foreign body in the nose, or growths inside the nose. Immediate management Your condition is as we all know, seasonal allergic rhinitis, hay fever. Long term intranasal steroid is the treatment of choice in treating this condition. (key point) So | will prescribe an affordable brand for you. If long term treatment is not alright for you, | can adjust to the types that can be used when necessary. (Beclomethasone and fluticasone over a short term 4-6 weeks). And also | will prescribe the lowest dose possible just to control the symptoms. Usually, unlikely to systemic steroids, which means steroids by mouth or injections, the serious side effects are not seen with nasal steroids, So it is relatively safe to use. | will also demonstrate the correct method of using it later. (Failure of treatment is common with incorrect usage, spraying upward to Little’s area leading to dryness, epistaxis) Your nasal decongestants it is not good for you in long term. It is only meant for a temporary relief. In long term, it may even worsen the runny nose symptoms which we call rebound phenomenon. So please stop using.(key point) Instead, you can try saline nasal spray 2-4 times daily which will also relieve the blocked nose and clear the mucus. It also enhances the effect of nasal steroids if do 15mins before steroids. For antihistamines, you can have it, when necessary, like when you have sneezing or epiphora (red, watery eyes)(key point) Further investigations -refer to allergen specialist for skin prick testing or allergy blood test (RAST) to know your triggers.(key point) Once you know them, you need to avoid them which is in important part of treatment. | will also run baseline blood tests, Blood counts as certain blood cells like eosinophils are increased in allergic condition and inflammatory markers, Further management “1 will follow you up after allergy test, then you'll have to avoid them, adjust your steroids regularly according to the symptoms, if not relieved with these measures, | will refer you to an immunologist/allergist for immunotherapy or if inferior turbinate hypertrophy (enlargement of nasal bones) referral to toan otorhinolaryngologist for turbinate reduction may be indicated 20 footing 1 Sabet 7 Chae wpa 3 kom / contrabie wh A Using RIGHT hand ‘ a v fetes \ ) toate mm, (eA DD) eo (ke /, ry 4 extol St coner vero i | 8 Curran rept cber a » : 6: Donets 4 8. Ear examination Young lady with 2nd time left ear pain. Tasks: History, PE, Dx and DDS plus if there is deafness then add dds of ear pain, such as acute otitis media, otitis externa, wax, injury, H. zoster vesicles, Trauma, intracranial mass etc. «inspection palpation Otoscopy hearing tests (there are different methods apart from the following) if you have time -cervical and supraclavicular LN -Cranial Nerve 7 -nose and throat Inside: | am so sorry to hear about your pain, do u need pain killer? pain Q- SOCRATES. Positive findings + 2nd time of left ear. + Throbbing pain, + no discharge, + no other positive findings about pain. hearing loss in left ear. No nasal discharge, Throat pain, neck pain, headache, eye pain or discharge 21 Footman amentcad last ear pain attack - took medicine and gone. Nil on present medical history including medicines, past history and family history. PE will look at your ears, feel and do some special tests (On inspection, (with comparing to right side) + no redness, obvious discharge, trauma, vesicles, battle signs or scars. Feel: no positive findings on palpation on It and rt ear Hearing test Rinne test — vibrating tuning fork will be put behind your ear and once u stop hearing pl let me know so | will bring it in front of ear and u need to let me know which one u can hear better. + Positive was left side BC>AC. weber tests: *+ positive was she can hear better on affected side. Classic left side conductive deafness. Now | am going to look inside of your ear, if you feel pain then pl let me know we will stop there. Will change the black tip and choose the widest possible. First | hold the otoscope with It and to inspect in left ear. started from outer ear, no vesicles ete. + Positive finding was red tympanic membrane without perforation and discharge. Dx: | think you have an inner ear infection we called acute otitis media, features are ear pain, hearing impairment in repeated cases, perforation of ear drum and discharge. Other less likely possibilities are + cholestetoma, which is’a skin growth in ear + wax (but I did not see any), + infection of outer ear ( but symptoms does not support it), + otosclerosis which is a growth of small bone in ear (less likely as it does not cause pain). Hearing test 1-whispering test + the number 68 is used to test high tone and 100 is used to test low tone. + at about 60 centimetres from the patient's ear. + Ifpartial deafness is suspected, perform Rinné’s and Weber's tests: 2- Weber test (is the buzzing louder on one side?) a vibrating 512 Hz tuning fork is positioned on the centre of the forehead. Nerve deafness = better in the normal ear. conduction deafness = louder in the abnormal ear 3-Rinné sx test (where does it sound louder?) a 512 vibrating tuning fork nerve deafness = air and bone conduction are reduced equally so that air conduction is better (as is normal). 22 Footman amentcad This is termed Rinné-positive. conduction (middle ear) deafness = Bone conduction is better than air condu: This is termed Rinné-negative. Causes of deafness. Unilateral nerve deafness may be due to: (1) tumours, such as an acoustic neuroma; (2) trauma, such as fracture of the petrous temporal bone; or (3) vascular disease of the internal auditory artery (rare) Bilateral nerve deafness may be due t () environmental exposure to noise; (2) degeneration, such as presbyacusis; (3) toxicity, such as aspirin, gentamicin or alcohol; (4) infection, such as congenital rubella syndrome, congenital syphilis; or (vi) Méniére’s disease. Brainstem disease is a rare cause of bilateral deafness. Conduction deafness may be due to: (1) wax; (2) otitis medic (3) otosclerosis; (4) Paget's disease of bone. Global score: 6 Key steps: allyes History: 5 Choice and technique of examination, organisation and sequence: 6 Accuracy of examination: 6 Dx/ DDs: 6 Performance guideline FDD NW woo MM ATP 1-Ear pain questions -Duration (already mentioned in the stem) - where exactly the pain is? Any pain in the other ear? -Radiation severity. -character. -anything make it better or worse? -has this happened before? 2-FDD NW -any fever? -any ear discharge? colour? -any hearing problems? One or both ears? Do you hear better in noisy environment? Have you 23 Footman amentcad increased the TV volume? -any nausea or vomiting? -do you feel room spinning around you? (Vertigo) 3-WO00 MM ATP wax: have you had problems with wax in your ears in the past? Otitis media: have you had recent infection? Any recurrent ear infections? Otosclerosis: is there a history of deafness in your family? Occupation: does your job or hobby expose you to loud noise? Meniers disease: any ringing in your ears? Medication: do you take any medications that affected hearing? Acoustic neuroma: any problems with walking or balance Trauma: any injury to your ears past medical history: have you had any history of serious infections like meningitis? 9. Cholesteatoma Ear Pain in a young man. Tasks ‘© Hx not more than 2 mins, ‘Physical examination, * diagnosis to the patient, Positive findings + painin right ear. + Left ear was fine, + The pain was deep in the ear, not going anywhere and increasing. + Nothing helped to relieve or aggravate it. + Itwas followed by ear infection although he didn’t have any discharge. had some hearing loss. + couldn't tell if he heard better in a crowded or noisy environment + increasing the volume of TV + had a history of ear infections previously for which took antibiotics. + was fine between episodes. + didn’t swim a lot. No air travel recently. No use of ototoxic drugs or exposure to loud music. No FH of deafness. + Non smoker and non drinker. Positive findings on PE. ‘+ examination on the mannequin, whose left ear was exposed. ‘+ compared with other ear. ‘© Palpated ear, parotid and mastoid. 24 Footman amentcad '* Held the otoscope in left hand and started inspection by commenting that ‘+ pulling the pinna upward and backward to straighten his ear canal. © There was no abnormality, no signs of trauma, no redness or swelling or any discharge. No swelling in-front of ear and looked for any signs of mastoiditis. © No wax, discharge or any foreign body was present. © Tympanic membrane was ruptured in the Attic area and there was deposition of white pearly material in the tympanic membrane. © TM was grey in colour with normal light reflex and handle of malleus. Steps summary: 1-Consent: | need to examine your ears this would involve having a look, feel and using some instruments would that be alright? 2-inspection or roleplayer: -shape and deformity -scars and swelling (always look fron and bet -redness and discharge, vesicles 3-palpation on roleplayer tenderness -swelling -LN (pre and post auricular) “if you want to pull the ear take consent-> | am just gonna pull your ear is that ok? \d the ear) 4-Otoscopy -When you use otoscopy take consent: Now I’m just gonna uses this instrument called otoscope to look inside your ears, checking the ear canal and the ear drum. | will be as gentle as possible if you feel uncomfortable at any time please let me know. Inspect ear canal: -redness, discharge, swelling, wax, foreign body. (in one case you will see lots of wax which will be your most likely diagnosis as you can’t see the ear drum clearly). -Inspect the eardrum: -redness, bulging, retraction, perforation, light reflex. (in one case there will be intense red TM +/- bulging with fluid behind it so your most likely diagnosis will be otitis media with/without effusion; infection of the middle ear with pus accumulating behind the ear drum) (in another case the there will be perforation in the unsafe zone of ear drum with deposition of whitish materials so cholesteatoma will be your most likely diagnosis; because of repeated ear infection you have perforation of the ear drum that happens in the upper part of the drum or the unsafe zone we call unsafe perforation. This lead to development of cholesteatoma, which is an abnormal skin growth in the middle ear behind the eardrum. When this growth increases in size, it can destroy the delicate bones of the middle ear causing the hearing loss and when infected, it can lead to ear pain and discharge. | will refer you to the ENT specialist for further treatment. You will undergo surgery with removal of the mass. The specialist might decide to do a CT scan and audiogram (hearing test) before deciding on surgery. 25 Footman amentcad S-hearing tests (there are different methods apart from the following) -whisper test (60 cm behind the patient say and number 55 or 99 any you like) | am just gonna stand behind you and whisper a number or a word. Is that ok. Please tell me what Ihave said, -tuning forks When you use tuning fork: | am just gonna use these tuning forks to test your hearing. choose 512 HZ weber test ( am putting this on the middle of your forehead tell me if the buzzer is louder on one side or the other side or in the middle. Rhinee test (Iam gonna place it once on a bony part behind your ear then in front of your ear and tell me where does it sound louder?) (I think in all cases there will be conductive hearing loss so whisper test positive on the left ear. Weber lateralized towards the left ear. Rhinee; left ear bone conduction> air condution. 6-if you have time -cervical and supraclavicular LN -Cranial Nerve 7 -nose and throat -Ask to touch the chin with chest to rule out neck stiffness in meningitis, explanation because of the repeated infections he had a ruptured ear drum in the upper part which is known as ‘unsafe perforation’ being in that sensitive part of ear drum. more concerned because he also had developed a condition called Cholestetoma, due to abnormal deposition of skin as a result of repeated ear infection. had some complication where it involved nerve that supplies the muscles of facial expressions and could also cause infection of the covering of the brains. Management (if asked) - Most likely you have a condition called cholesteatoma and a conductive hearing loss. It is an abnormal skin growth in the middle ear behind the ear drum. When this growth increases in size, it destroys the delicate bones of the middle ear causing the hearing loss and when infected, it can lead to ear pain and discharge. - | will refer you to the ENT specialist for further treatment. You will undergo surgery with removal of the mass. The specialist might decide to do a CT scan and audiogram (hearing test) before deciding on surgery. | treatment will be careful cleaning of ear (ear toilet) and some topical an like ofloxaci drugs 26 Footman amentcad The main aim of the surgery is to remove cholesteatoma and create a safe and dry middle ear. - Advise on risk factors: smoking - Later on, you can undergo a second surgery for reconstruction of damaged middle ear bones. ~ This is potentially serious as it can lead to facial nerve injury and intracranial extension such as brain abscess and meningi - Review regularly every 3-6 months. - Reading materials. Notes Summary there are 3 cases for ear examination 1-Choleasteatoma 2-otitis media with/without effusion 3- wax Regarding history: is the presenting complaint is ear pain do like above. If hearing loss then start from hearing loss questions then ask if any ear pain and continue like above So pain questions + FDD, NVV+ WOOO MM ATP 10. Testicular torsion ‘An 18-year-old boy complained of pain in the right groin and vomiting for 3 hours after riding a bicycle. Tasks * h/o © PEFE © Dxand Mx positive findings h/o biking then sudden pain in lower tummy for 1 hour sudden onset unilateral testicular pain and swelling pain mostly constant 9/10 pain in iliac fossa ass with NV PE unwell, distress pt tachycardia discoloration of hemi scrotum very tender and swollen high riding/ horizontal testis cremasteric reflex absent phren’s sign negative 27 Footman amentcad pbx © Testicular torsion * Trauma + Strangulated hernia * ut * Renal stones * Appendicitis + Epididymo-orchitis * Intestinal obstruction feedback ask sexual activities 11, MMSE, You are a General Practitioner and a patient Alex came in your surgery with a history of chronic alcohol use of 6 SD/day and bs! is 12mmol/L. Task Explain what MMSE is to the patient Perform a complete MMSE Interpret and explain the normal and abnormal results to the patient and explain its’ significance Positive findings ration and recall problem Performance guideline Explain MMSE: Today | am going to do a test called Mini mental state examination which is a simple bedside screening test to assess your memory and thinking. In here, | am going to ask you some questions and give you some problems to solve. Please try to answer as best you can. It will assess u in number of areas and has 30 points in total. It will hardly take 5 mins & | will guide you through out the test. If at any point you have any question or you don’t understand me, please don’t hesitate to ask me. Is that alright with u? 28 5 ‘What is the yoar? Season? Dale? Day ofthe wook? Month?” 5 “Where are wa now: State? County? Towneity? Hospital? Floor? Tho sxaminar ramos three unrelated abjecta Cleary and slowly, than 3 _asks the patient to name all three of them. The patient's response is used for Scoring. The examiner repeats them until patient leams all of thom, if possible. Numbor of tials" “Lwould lika you to count backward tram 100 by sevens.” (03. 86, 79, 8 72, 65, ...) Slop after fe answers. ‘Ailernative: “Spell WORLD backwards.” (D-L-R-O-W) “Earler | told you the names of tree things. Can you tell me whal those 3 were?” > ‘Show tho pationt io simplo objects, such as a wristwatch and a penal, and ask the patient to name them. 1 “Repeat the phrase: ‘No is, als, or buts. 3 “Take the paper in your right hand, fold i in hall, ard put iton the floor.” | (The examiner gives the patient a piece of blank paper) 1 “Please read this and do what it says.” (Written instruction is “Close your eyes.") ; “Make up and write a sentence abou! anything.” (This sentence must contain a noun anda verb.) “Please copy this picture" (The examiner gives the patent a blank piece of paper and asks him/her to draw the symbol below. All 10 ‘angles mus! be presert and two must intersect) , CR z aa PERFORM MMSE: CCOMPONENTS OF MMSE: oraric (30) 1. Orientation 2. Registration 3. Attention 4. Recall 5. Language 6. Construction QORENTATION UTime: (5) 1. Year 2. Season 3. Date 4, Month 29 Footman amentcad 5. Day of the week Uplace: (5) 1 State 2. City 3. Suburb 4. Street 5, Street Number/ Floor 1. Allow ten seconds for each reply. Say: a) What year is this? (accept exact answer only) /1 b) What season is this? (during the last week of the old season or first week of a new season, accept either) /1 ) What month is this? (on the first day of a new month or the last day of the previous month, accept either) /1 ) What is today’s date? (accept previous or next date) /1 ) What day of the week is this? (accept exact answer only) /1 2. Allow ten seconds for each reply. Say: a) What country are we in? (accept exact answer only) /1 b) What state are we in? (accept exact answer only) /1 c) What city/town are we in? (accept exact answer only) /1. d) What is the street address of this house? (accept street name and house number or equivalent in rural areas) /1 What is the name of this building? (accept exact name of institution only)/1 e) What room are we in? (accept exact answer only) /1 What floor of the building are we on? (accept exact answer only) /1 CIREGISTRATION | am going to name three objects. When | am finished, I want you to repeat them. Please Remember what they are because | am going to ask you to name them again in a few minutes - APPLE - COIN - TABLE Please repeat the three items for me (score one point for each correct reply on the first attempt) 3 Allow 20 seconds for reply; if the person did not repeat all three, repeat until they are learned or up to a maximum of five times (but only score first attempt) ATTENTION: Spell the word WORLD backwards please (allow 30 seconds; if the person cannot spell world even with assistance, score zero) “DLROW" RECALL: Now Please repeat the three objects | asked you to remember LANGUAGE OPointing and naming of two objects: Show pen and watch (2) Oepetition of sentence: Can you repeat a phrase after me? 30 Footman amentcad “No if's and’s or but’s.” (1) Uhre stage command: Take the paper on your right hand, fold the paper in half and put it in your lap (3) ‘Ask the person if he is right or left handed. Take a piece of paper, hold it up in front of the person and say the following: Take this paper in your right/left hand (whichever is non-dominant), fold the paper in half once with both hands and put the paper down on the floor. Reading and obeying: “Close your eyes.” Show the paper and ask them to read and follow the written command, (1) hand the person the sheet with CLOSE YOUR EYES (score on reverse of this sheet) on it. if the subject just reads and does not close eyes, you may repeat: Read the words on this page and then do what it says, a maximum of three times. Usentence Writing: Would you please write a sentence for me? (1) CONSTRUCTION: Copy this design please. Allow multiple tries Impaired Registration and Recall: Dementia Impaired Orientation and Attention/Calculation: Delirium Oxplain the results: Explain both positive and negative findings. You have problem with registration learning new things) and recall ( remembering it) . It signifies you have short term memory loss. It could be due to many reasons: - Alcohol induced brain injury - Vitamin B12 deficiency - Electrolyte imbalance - other metabolic such as thyroid dysfunction , diabetes neuropathy - infection such as Meningitis (least likely } - nasty growth in the brain such as Brain Tumor - Head injury - Early Dementia 12. Threaten abortion with hyperemesis gravidarum Your next patient in ED is a 37-year-old primigravida with bleeding PV since yesterday, confirmed at home by urine pregnancy test. Since last week, she has been troubled with excessive nausea, vomiting as well. Tasks History PE Explain Dx with reasons Discuss Mx to pt Positive findings 31 Footman amentcad - 2 pads per day for 2 days, mild bleeding, no clot, bright red color, no other discharge, no tummy pain - Nausea and vomiting +, severe and can’t keep down anything, can’t eat at all - LMP 8 wk ago - PT at home done and + - Reduced urine output - Married, no STI - Tired and dizzy - No medical problem ~ No smoking nor drinking - Folic acid only, no other medication - No known drug allergy PE card = Dehydration +, Lethargic, no pallor - BP =90/50 - HR=110 per min - PaSO2=98% - Abd examinatior ad ~ Pelvic examination ~ single fetus — viable, ut size consistent with GA, CMT -ve, ~ No active biding, Cx os closed - _UDT-ketone body + - BS=normal Key point - Biding history + NVhistory (twin, molar, hyperemesis gravidarum) - Dehydration +ve, hypotension - WVline, take out bid, grouping and matching, fluid = Admission = Specialist will see - Anti-emetic medication ~ _USno need as per pelvic examination findings — can’t be ectopic, incomplete miscarriage - _invnecessary - bid for baseline and Urea/Cr/El, Urine sample 13. GBS 4ve in pregnancy ‘A.25-year-old lady with 10" week pregnancy came to your GP for the result. You saw her last week as her first AN visit and blood tests, urine test was done. All the results are normal, except urine test shows GBS + in mild stream urine sample. Tasks: -explain results to pt 32 Footman amentcad -take relevant history -discuss Mx to pt Positive findings - Planned preg - No discharge nor bleeding ~ No fever, no abd or back pain - NoUTI symptom - First time, no other medical problem - Condom used before - Married, no STI ~ No smoking nor drinking - No known drug alleray ~ Only folic acid is taking Feedback - UTI symptoms - Kidney problem, Pyelonephritis and stone h/o - 0M ~ Stable partner - stl ~ Regular ANC? - Urine C&S ~ AB for a week (check drug allergy) ~ cephalexin (no trimethoprim during preg) - Review after 2 days to see progress and according to the result ~ may need to change AB - After complete AB — recheck urine if there is any infection - Usually, GSB + is not serious and common in women however in preg it needs attention as it can catch to baby and can proceed to serious infection to mother due to reduced immunity during pregnancy - Will arrange for USG and down screening - Personal hygiene - Red flags — fever, tummy pain, UTI symptoms, blding/ discharge = need to come back 14. Advanced age pregnancy 42 years old woman comes to your GP clinic after her Home pregnancy test appears positive tasks history -PEFE card -counsel the patient Positive findings «Planned pregnancy © LMP - 6 weeks ago, regular, no pain 33 Footman amentcad «Stable partner, no STI + Frist pregnancy, no miscarriage, no discharge nor bleeding OCP and stopped since a year ago «Pap smear up to date Other systematic — normal PEFE card © everything normal except BMI 32 Performance guideline ‘ory 1-approah -Ican see that you are here for checkup as your HPTiis positive. Isit a planned pregnancy? -congratulation! 2-5Ps questions periods -when was your LMP? -were they regular? -any pain or heavy bleeding during periods? Partner -are you in a stable relationship? -is your partner supportive? -have you or your partner ever been diagnosed with STI? Pregnancy this your first pregnancy? -have you had any previous miscarriages? -do you have Nausea, vomiting or breast tenderness? -any tummy pain, vaginal bleeding or discharge? Pills -what contraceptive method were you in? -how long have you been off the pill? pap or HPV -is your pap or HPV up to date? 3-General questions -do you take any medications? -do you start taking folic acid? -Past medical history (HPT, DM, SLE, Epilepsy, heart) -SAD (does not smoke or drink) family history of birth defects? -diet and vaccination? -blood group? PEFE card 34 Footman amentcad Counselling finish examining you let me assure you that everything looks fine. | could not find any serious problems. -your vitals are normal. Your heart is fine and tummy as well. There is one thing, which is your BMI which is thing we use to assess your weight which appears to be higher than it should be. Normally it should be less than 25 but yours is 32. is good that you are here so that | can discuss about your pregnancy. firstly, | need to confirm your pregnancy by doing an office pregnancy test. -I would also like to arrange some routine blood tests for you: FBE, Blood group and RH, BSL, UCE, urine MCS, LFT, vitamin D, serology for chicken pox and German measles, and ST! screen with your consent. -you need to follow some life style measures: * take a healthy balanced diet rich in fruits and vegetables, cereal and bread,, avoid soft cheese, raw meat, *increase your fluid intake “limit coffee to 2 cups a day. | appreciate that you stop smo} for your pregnancy. * do regular non-contact exercise “Ican also refer you to a dietician Will help you attain the ideal weight. - | would you to continue with your folic acid but | will prescribe a higher dose, to be taken in the 1° 3 months of the pregnancy. 1g and drinking alcohol as it is good -Because of your advanced age, | need to refer you to high-risk pregnancy clinic where you will be seen by a specialist and will have frequent regular ANC checkup with regular visits up to your delivery. -during each visit, your body weight and BP will be recorded along with baby wellbeing, -let me assure you that many woman at your age can go through normal pregnancy and have safe delivery. But for further precautions we offer close monitoring as there is a risk of having ‘complications during pregnancy with increasing age for example: DM, high BP, PROM, Preterm,. Any of these if detected can be managed properly by specialist at high-risk clinic. for the baby there can be a risk of birth defects and the most important one is Down syndrome. So that we offer screening tests at 11-13 weeks of pregnancy which is a combined test; blood tests and us in addition to confirmatory test in the 1* or 2" trimester; CVS in 1* trimester when a needle inserted through the tummy to take a sample of cells from placenta. And amniocentesis in 24 trimester; when needle passed into your tummy to take portion of fluid in the bag of water surrounding the baby. Risk of miscarriage with CVS is 1% while 2% for amniocentesis. -you will have Ultrasound imaging at 18 weeks and repeated ultrasound at 32 weeks, sugar test at 28 weeks and bug test at 36 weeks. -lots of things but Do not worry | will give you reading materials about all of these. -red flag. (Tummy pain, bleeding, discharge) -review. 35 Footman amentcad 1S, Pre pregnancy DM counselling AMC Case (9-2-2018/ 1-6-2018/ 6-6-2018) ‘Woman Type 1 DM on insulin. Plan to become pregnant. Her husband is a farmer and they are living in a rural area 2.30 hr from tertiary hospital. Would like to conceive but wants to have care and delivery in local rural hospital. Tasks: -take history -PEFE -convince to have ANC is tertiary hospital. «planning for the pregnancy in 6 months diagnoses with DM since 9 years old «currently no symptom of DM * to control you DM — Insulin, taking regularly + checkup - (7 months ago and HBAIC is 11%) + Periods: LMP, regularity (2 weeks ago and regular) partner: support, STI? (Normal) pregnancy: 1* or not, previous miscarriage (1") pill (No) Pap or HPV (up to date) reason for why you want to deliver in rural hospital - | wants my husband and family members to stay around me ‘+ urine dipstick (glucose 2+) Performance guideline ‘ory 1-Approach -Hi my name is I'm one of the doctors here | understand from the notes that you have DM and you've come to discuss about pregnancy planning. Is that right? -when are you planning for the pregnancy? (in 6 months) is a great start from you to come here and discuss about it. So we can address it together. t, can | just ask you a few questions to assess your current health condition? jabetis Mellitus questions since when have you been diagnoses with DM? -do you feel thirsty? Are you passing large amounts of urine? Any trouble seeing? Any skin or vaginal infections? 3-Medications questions -what medications do you take to control you DM? -do you take it regularly? 36 Footman amentcad 4-Well control questions -have you had regular check-up with your physician? When was the last one? What was the result? -do you regularly measure you BSL at home? What was the most recent result? -is there any time when your BSL was too low or too high that you had fainting episodes or need hospitalisation? 5-5Ps questions “Periods: LMP, regularity -partner: support, STI? -pregnancy: 1" or not, previous miscarriage -pill -pap or HPV 6-investigations -have you had any blood tests like (blood group, rubella or varicella serology) 7-May | know the reason why you want to deliver in rural hospital? Physical findings from examiner 1-GA (PODL) 2vs 3-BMI 4-chest and heart: air entry, breathing sound, dullness, adventitious sounds, apex beat, heart sounds and murmurs. 5+ focus abdomen inspection: distension, mass palpation: mass, tenderness auscultation: bowels sounds 6-pelvic inspection of vulva and vagina: discharge, bleeding, rash, vesicles speculum: healthy cervix, discharge Bimannual: uterine size and tenderness, adnexal mass and tenderness. T-office tests -urine dipstick -BSL Counselling Effect of DM on pregnancy (mother and baby) ‘* assure you that many woman with diabetes can have normal pregnancy ‘+ can get through safe deliveries with healthy babies. © However, you need to be closely monitored during pregnancy as there are some issues that might arise during pregnancy. ‘© DM can increase BP during pregnancy or increase amount of fiuid in the womb called polyhydramnios. ‘Having DM can increase the risk of birth defects or having a big baby, it can cause breathing problems to the baby. 37 Footman amentcad ‘+ However, these risks can be minimised by keeping your BSL during pregnancy under control in addition to monitoring you closely with the help of MDT. © Soif any complications happened it can be dealt with appropriately. Prepregnancy from history and examination it seems like your blood sugar is not well controlled so | would like to perform some blood tests like BSL, HBAIC to assess DM control. | would also like to refer you to diabetic physician for complete assessment of nerves and kidneys and your DM refer you to eye specialist to check your eyes. once all investigations are normal then it will be safe to attempt pregnancy. | will start you on folic acid now, (2.5 mg high dose)to be taken prior to pregnanct and continued for the 1* 3 months of pregnancy. And will also arrange all routine blood tests usually performed at 1* antenatal visit. During pregnancy you will be managed at high-risk pregnancy clinic by MDT you might need increase In insulin requirement to keep BSL under control. need to go through more frequent antenatal checkups Delivery will be in tertiary hospital under specialist guidance. Usually planned at 38-40 weeks After delivery insulin dose will be return back to Prepregnancy level. you can breastfeed your baby. ‘Address rural concern lunderstand that the presence of your husband or family members is very important and comfortable to you. the reason why you need to be managed in the tertiary hospital is that rural hospitals have limited facilities as well as specialists, and due to all possible complications that can happen during pregnancy like big baby, high BP, PROM it would be best for you and your baby to be managed in tertiary hospital where facilities are available there. Feedback ~ passed with all 5 Feedback 1-6-2018 Pre preg counselling. 12 DM. Task: Hx, Counseling 6 months ago last HbA1c 11%. Presently RBS 8. Under insulin. Performance guideline © Asked DM questions, © pre preg questions. 38 Footman amentcad ‘+ Explained need to repeat HbAtc, ‘* complications in preg and what to expect, * increased insulin demand ete. Positive findings ‘© Type 1 DM on insulin. ‘* Plan to become pregnant in 6 months. ‘+ husband is a farmer and they are living in a rural area 2.30 hr from tertiary hospital. ‘* She wants to deliver at rural hospital © She was dx since 9 yo * so farno hx hospitalised due to her illness, routine checking up her BSL, «last time to see sp 3 weeks ago all were normal , * blood test normal eye normal, no bowel/urinary problem , no weight loss no vaginal problem no hx of STI in stable relationship. Partner supoortive , «living in rural want to be deliver in local hospital , * papsmear were normal never pregnant before didn’t take any contraceptive ‘© SADMA only diabetes + finding © LastHbAIc8. ‘© last specialist visits 7 months ago. On Ex all normal except urine sugar2+. It was a confusing station as she didn’t have good control pts wanted to deliver in rural set-up Blood gp B +ve, no other chronic ailments. she wants her family members to stay around her and no financial and other issues PEFE: stigmata of dm complications - normal * Mx | don’t think it’s the right time to conceive as well controlled DM itself make a pregnancy high risk for mum and the baby and in your situation ur blood sugar is not controlled ‘first we need to run fresh set of investigation, ‘© seek specialist review for DM management and adjustment of insulin accordingly, maintain DM diary and started her on folic acid then told her compli of dm on pregnancy , risks to her and baby its comforting to see all the loved one around you once u r delivering the baby but because of high-risk issues and complications | would suggest you to review your decision and arrange family meeting with ur consent as she needs frequent monitoring during pregn and at the delivery even after delivery there are risks for baby like RDS. I said how do u feel about mx plan she said | will do whatever you will suggest 39 Footman amentcad ‘* Isaid good lets work as MOT (referral to DM physician, high risk preg clinic ince she is pregn, but ran antenatal tests, and dietitian referral) Global score: 4 © Approach to pt: 3 oH 5 Choice and technique of exam :5 Pt education: 5 16. Thyroid examination Case 1 (Hyperthyroidism) Your next patient in GP practice is a 27-year-old lady with restlessness and weight loss. The patient is on propranolol 20mg for prevention of migraine headache. The migraine is well controlled -Perform relevant physical examination -explain your findings to the patient. -tell the possible differential diagnoses. Positive findings GA- thin and restless Nail - white polish to mislead you about leukonychia so say just polish and cannot elicit nail changes warm, sweaty = positive Fine tremor + (hyperreflexia +ve) Examination steps (talley @ connor) 1-WIPE 2-General appearance (in exam the patient will be thin and restless) -anxious -restless-thin -weight. 3-Hands look and feel together nails (clubbing, Onycholysis) in the exam there will be a white polish to mislead you about leukonychia so say just polish and cannot elicit nail changes “do clubbing test by put both hand’s index finger upside down against each other and see if there isa space or no, -palmar erythema -warm, sweaty (will be positive) “Fine tremor ask patient: if you could bring your arms in front of you like this and splay your fingers. | am just 40 Footman amentcad gonna lay a piece of paper on your hands. (this will be positive) -pulse (rate and rhythm) 4-Face (in exam the patient may pretend to make eye signs but you need to decide if it looks real then say it if not say could be) look -thyroid stare (itis like patient staring on you with all eyes open) -ptosis (drooping of eyelids) -proptosis (protruded eye). -Exophthalmos (I think if the lower sclera was obvious and not covered by the lower eyelid) -lid retraction (I think when the upper sclera was obvious and not covered by upper eyelid) -conjunctivitis -corneal ulceration Move (all -ve in the exam) slid lag *keep your head still and follow my finger. -eyes movement (opthalmoplagia) (like H shape movement) ask patient if blurring of vision present, 5-Neck (all negative in the exam but sometime they give real patient) look “scars -dilated veins -skin redness -swelling -ask patient to swallow water “if you could just take a sip of this water and hold it in your mouth...and swallow. -ask patient to protrude her tongue "if you could just stick your tongue out. Feel (from behind) flex the neck slightly and start palpating the mass *can you flex neck slightly forward and relax. site, size, surface, regularity, consistency, tenderness, temperature, mol -ask patient to swallow water and repeat palpation palpate the inferior border of the mass. -palpate Lymph nodes tracheal position (from front) *now | am just gonna feel the windpipe it might be a little bit uncomfortable. Percussion (tap on the upper part of sternum 2 sites with your fingers horizontally over the sternum) Lam just gonna tap on your chest. (retrosternal goiter}. Auscultation -over each thyroid lobes (bruit) -stridor (apply mild compression on the lateral lobe) no need if there was no swelling. Special pemberton signs for retrosternal goiter. can you raise your hands above your head like this. a Footman amentcad 6-Chest look: gynecomastia Feel: apex beat listen: heart sounds and murmurs T-others -reflexes (biceps) (hyperreflexia +ve) (may be a cr -pretibial myxedema -proximal myopathy (leg): *can you cross your arms and stand up for me. iagnosis and differentia diagnosis 1-hyperthyroidism (most likely): thyroid gland is over active with increase hormone secretion causing your symptoms. 2-graves disease (autoimmune disease) (young woman with eye signs and thyroid bruit) thyroiditis (infection or inflammation) less likely no tenderness, warmth or fever. a-anxiety 5-medications less likely as she does not take any such medications. 6-nasty growth or cancer less likely from examination. al error if we did not examine the reflex) Feedback 21-6-2018 Nervousness: Pass:/Global score:4 Your next patient in GP practice is a 27-year-old lady with restlessness and weigh loss. The patient is on propranolol 20mg for prevention of migraine headache. The migraine is well controlled Perform relevant physical examination, explain your findings to the patient and tell the possible differential diagnoses. When | entered the room started to introduce myself, in the middle of my greeting, the role player asked: Aren’t you going to examine me?” and | said of course, if you don’t mind. Then | washed my hands and explained about the examination, and asked for consent, then started with general appearance: In general appearance, she is a young, thin and lean lady, she looks fidgety and restless, however she does not look cachectic or ill. On inspection of the eyes, | addressed that | cannot notice a real lid retraction or exophthalmos, in conjunctiva inspection, there was no pallor, as well no other signs of anaemia such as angular chelosis, or glossitis in the mouth. Then proceeded to the examination of the movements of the eye, which were normal and no lid lag was present. On inspection of the neck, there was no swelling. | explained to the patient that I need to stand behind her and palpate her neck for the examination of the thyroid. | could not feel the thyroid gland, as it was hardly palpable, and there was no lymphadenopathy in the neck. Also no bruit in the auscultation of the thyroid gland. During the examination | asked the patient to swallow her saliva twice and palpated each side of the gland. Then proceeded to the auscultation of the heart. I listened to four spots on the chest, and mentioned that | can hear $1 and S2 clearly, without any added sound. Then the examiner said heart is normal. Then | asked the role player to get up from the chair without using her hands or arms, and sitting again, there was no signs of proximal myopathy. I checked her legs and feet for pretibial myxedema. then proceeded to hands, checked her pulse, | did not count the pulse, but mentioned that pulse rate is regular and its count looks like being around 70 to 75, however because the patient is taking propranolol, I do not expect to find tachycardia. Then checked her tremor, (I had almost forgotten to check the tremor, the role 42 Footman amentcad player Guided me herself, by keeping her hands outstretched, there was a fine to slightly coarse tremor, in the hands, the hands were sweaty, but no palmar erythema, and no swelling. Her nails were done with white polish, (| do not know whether | was supposed to take that artificial nails as clubbing or not, however, | asked her to put her finger nails against each other, and mentioned that this was not a real clubbing, and to me it looks like a normal hand, without signs of thyroid acropachy. I also asked the examiner to give me a blood pressure cuff to measure BP, he said, it was 125/ 75 , also asked for medical hammer for checking the reflexes, | checked biceps reflexes on both sides which were obviously increased. In my differential and explanation, | explained that she does not show the full picture of, hyperthyroidism, however, some of these signs appear late during the disease course, and also as she is taking propranolol | do not expect to find anaemia or heart problems in her, then | added that by approaching nervousness or tiredness | can give several different diagnoses, but by considering all her symptoms together | cannot really think of any diagnosis other than Hyperthyroidism, which is caused by more than normal secretion of the thyroid hormones, so she needs to have a blood test so that we can exclude thyroid problems. Then the bell rang and lexited the room. 17. Threaten abortion with retroplacental blood clots ‘25-year-old lady with a 7 wk pregnancy presents with bleeding PV since yesterday. This is her first pregnancy. Tasks - History - PE = Request inv from examiner - Explain Dx with reasons to pt = Discuss Mx with reasons to pt ~ Planned preg, first preg ~ Bleeding for 2 days, 2-3 pads per day, bld clot, no tissue/grape like structure, bright red color, no discharge, tummy pain last night - Fever/ chill and rigor — nil - Noh/oUTl = Married, no STI = No h/o miscarriage - Home preg test done - ANC for 1 time, bld gp O+ - No blding problem, no medication - Just folic acid = No smoking or alcohol drinking - Use marijuana only one time 2 months back 43 Footman amentcad PE = In pain, vital signs are stable - Abd-NAD ~ Pelvic - introitus blding +, ut = 10 wk, os close, no CMT - _US~ fetal cardiac activities, retroplacental bld clot ~ 4 to 5 cm Feedback = Pain history explore - Usually threaten miscarriage not ass with pain so that if there is pain ask for illicit drugs use, ask for current use and offer for support to quit drug abuse - Specialist consultation due to there being retained blood clot although fetus are viable according to US 18. Retained placenta with shock You are on call EMO at the hospital. You are called to see a 30 year old lady who recently delivery baby at 39 weeks of preg. Delivery was unremarkable but placenta was not coming out. On examination BP ~ 80/50 mmHg, HR ~ 104/ min. 3" stage lasts for 2 hr. this is her 1* preg Tasks Explain Dx to pt Explain possible consequences to pt Explain mx to pt Dizzy and not feeling well Do not want to remove the uterus if possible feedback ~ retained placenta with shock due to blood loss because excessive blood loss can occur with retained placenta = normally placenta come out within 30 min ...in this case taking longer that’s what we call retained placenta - Call for help - IV, urgent Bld gp and matching and other bid test, Fluid infusion and will repeat as per necessary, invx, will check the delivery note to find out any issue - Possible cause - Bladder distension, floppy uterus ~ Will do— empty bladder, uterine message, syntocinon inj and through the fluid drip, controlled cord traction (up to 3 times) - fit doesn’t work, inform the specialist and may need to do EUA and manual reduction, uterine artery embolization and removal of ut as the last option 44 Footman amentcad 19. Hip Examination Hip osteoarthritis ‘A64 years old man with a history of pain on his right hip joint for the last 6 months comes to your GP clinic. The pain is worse with activity. He tried Panadol but did not get relieved from the pain. Tasks relevant hip examination+ running commentary -diagnosis -management Differential diagnosis ‘Losteoarthritis 2-rheumatoid arthritis 3-Trauma 4-greater trochanteric bursitis 5- avascular necrosis of femoral head (painful gait or limping) (+ve on the right side) delenburg gait (#ve on the right side) (unable to walk heels or toes.) (Trendelenburg test +ve on the right hip) Thomas test (Fixed flexion deformity +ve on the right side) -femoral head (distal to the mid point of inguinal ligament) (+ve in OA) -adductor tendon (inner side if the thigh adductor tendon) (+ve in adductor tendonitis) -greater trochanter (ve in trochanteric bursitis) (+ve tenderness on right femoral head) Active flexion (+ve ited) passive internal rotation; (when foot lateral) (+ve limited) true leg length (+ve short right leg) “SLR test (+ve) (<30 degree joint/ >30 degree sciatica) -FABER test (to rule out SU pain) (+ve). *+ve test is when there is pain or discomfort: if on hij OA/ if on back croiliac joint ation steps 1-Gait -if you could walk to the other side of the room for me the turn and walk back. Look for: 45 Footman amentcad antalgic gait (painful gait or limping) (+ve on the right side) Trendelenburg gait (+ve on the right side) -Now walk on your toes and then on your heels (stand beside the patient) (unable to walk heels or toes.) 2-Trendelenburg test -1 am just gonna assess the muscles surrounding your hip joints. -if you could stand up straight for me -1am gonna stand behind you and just feel the muscles of your hip. ‘you could lift your left leg for me (the unaffected one): test is +ve if the pelvic drop on the side with the foot off the ground (Trendelenburg test +ve on the right hip) Note/ for more understanding this patient has pain on the right hip so the problem is in the right hip joint. So the only support for walking is his left hip. When you asked the patient to lift the unaffected leg off the ground which means the left side then there will be no support and the pelvic will drop towards the left or the normal side; which means the right or the affected side elevate while the left or the unaffected side drop. But when you say +ve need to give where the problem is; mean for the right hip or right hip problem. 3-Look (good exposure standing front and back) (usually -ve) “Scars -skin colour changes -swelling -deformity (fixed flexion) -muscle wasting 4-Thomas test (Fixed flexion deformity +ve on the right side) -put your hand behind the back of the patient (the lumbar area) -if you could bring your left knee as far to the chest as possible and hold it afterwards. -test is +ve when the extended leg is lifted off the table -test the other leg. Note/ in this test you are looking for fixed flexion deformity that might be visible on inspection but sometimes not. So here ask the patient to lift the normal side here the left one all up and hold it by his hands. Then notice the right leg if they elevate or raise then the test is +ve as in this, case) 5-Feel (for tenderness 4 points) -ASIS femoral head (distal to the mid point of inguinal ligament) (+ve in OA) -adductor tendon (inner side if the thigh adductor tendon) (+ve in adductor tendonitis) -greater trochanter (+ve in trochanteric bursitis) (+ve tenderness on right femoral head) 6-Move (6 movements} Active flexion (+ve limited) -passive abduction -passive adduction -passive external rotation; (when foot medial) -passive internal rotation; (when foot lateral) (+ve limited) 46 Footman amentcad Note/ -when you do passive abduction or adduction; put one hand on the heel and the other hand on ASIS. -when you do ER or IR; put one hand on knee and the other hand on foot on flexed leg. 7-Measure (true and apparent) true leg length (+ve short right leg) -apparent leg kength (normal) Note/ true leg length= from ASIS to medial malleolus) apparent leg length= from umbilicus to medial malleolus. Difference in true leg length indicates hip disease on the shorter side Difference in apparent leg length indicates tilting of the pelvis. 8-Special tests -SLR test (+ve) (<30 degree joint/ >30 degree sciatica) -FABER test (to rule out SU pain) (+ve). Flexion of the knee then abduction then external rotation= figure of 4) *put the R leg on the knee of the left leg “apply pressure on the right leg as to push it down while one hand on the opposite ASIS. *+ve test is when there is pain or discomfort: if on hip= OA/ if on back=sacroiliac joint “test the other side 9-Conclude with joint above and joint below. 1gnosis and management -most likely have OA of the hip. It is resulting from wear and tear as a result of excessive use of your joint over the years or due to old injuries in the affected joint. -the cartilage that covers and protect the end of the bone gradually wears away causing the joint to become rough and stiff. -most cases are mild and with treatment you can cope with it well -other possibilities are trauma, RA on inflammation of hip joint due to autoimmune disease. No ‘enough supply to head of thighbone and etc... Investigations -I will arrange an X-ray of hip joint Treatment relative rest during acute pain 2-painkiller 3-crutches 4-weight loss S-refer to physiotherapy for strengethening of mucles 6-occupational therapist for walking aids Adductor tendonitis your patient in GP is a 20 year old man complaining of pain in the right groin. Tasks 47 Footman amentcad -Physical examination -diagnosis and management Differential diagnosis L-adductor tendonitis Zosteitis pubis 3-hernia 4-trauma 5-0A 6-refered pain from lumbosacral spine Examination steps 1Gait -if you could walk to the other side of the room for me the turn and walk back, Look for: antalgic gait (painful gait or limping) (-ve) Trendelenburg gait (-ve) -Now walk on your toes and then on your heels (stand beside the patient) (-ve) 2-Trendelenburg test (-ve) 3-Look (good exposure standing front and back) (~ve) “Scars -skin colour changes -swelling -deformity (fixed flexion) -muscle wasting 4-Thomas test (-ve) 5-Feel (for tenderness 4 points) -ASIS -femoral head (distal to the mid point of inguinal ligament) (+ve in OA) -adductor tendon (inner side if the thigh adductor tendon) (+ve in adductor tendonitis) -greater trochanter (#ve in trochanteric bursitis) (+ve tenderness on right medial groin) 6-Move (6 movements} Active flexion -passive abduction -passive adduction (+ve painful) -passive external rotation; (when foot medial) passive internal rotation; (when foot lateral) ‘7-Measure (true and apparent) (-ve) true leg length -apparent leg length 48 Footman amentcad 8-Special tests -Squeeze test (+ve) *ask patient to flex both knees and hips “put your fist between flexed knees “ask patient to push on your fist. *+ve test when there is pain. 9-Conclude with joint above and joint below. Explain diagnosis and differentials -most likely have a condition called adductor tendonitis also called groin strain which is when the tendons near the groin become stressed and tensed and can tear causing pain -common in sport activities. -other possibilities are... Investigation X-ray and USD to rule out pain from bone or tendon. Treatment -avoid activity until pain settled -apply ice every 3-4 hours for 20 minutes until pain free -compress the thigh with help of elastic bandage or tape. -Pain killers -physiotherapy -not relieved-- steroid injections -come back if pain persists Trochanteric bursitis ‘A 45 year old female complaining of pain in the right outer hip that travels down her leg since last week. Tasks -Physical examination -diagnosis and management ferential diagnosis trochanteric bur 2-osteoarthritis 3-Trauma 4-iliopsoas tendinitis, S-avacular necrosis of femoral head 6-lumbar spine radiculopathy Examination steps 1-Gait -if you could walk to the other side of the room for me the turn and walk back. Look for: antalgic gait (painful gait or limping) (+ve) 49 Footman amentcad Trendelenburg gait (+ve) -Now walk on your toes and then on your heels (stand beside the patient) 2-Trendelenburg test (+ve) 3-Look (good exposure standing front and back) (~ve) “Scars -skin colour changes -swelling -deformity (fixed flexion) -muscle wasting 4-Thomas test (-ve) 5-Feel (for tenderness 4 points) -ASIS femoral head (distal to the mid point of inguinal ligament) (+ve in OA) -adductor tendon (inner side if the thigh adductor tendon) (+ve in adductor tendonitis) -greater trochanter (ve in trochanteric bursitis) (+ve tenderness on greater trochanter) 6-Move (6 movements) Active flexion -passive abduction (+ve painful) -passive adduction -passive external rotation; (when foot medial) -passive internal rotation; (when foot lateral) 7-Measure (true and apparent) (-ve) -true leg length -apparent leg length 8-Special tests “SLR (-ve) -FABER (-ve) -Squeeze (-ve) 9-Conclude with joint above and joint below. Explain diagnosis and differentials -most likely have a condition called trochanteric bursitis which is hip pain caused by inflammation of fluid filled sac called bursa on the outer edge of the hip. -can results from falls, trauma, overuse. -other possibilities are. Investigation (to rule out other possi -X-ray and USD Treatment -you need to rest and reduce activity for a few days. -put ice pack on the painful side -painkillers to relieve the pain. -try to avoid sleeping on the affected side -you can use a small pillow to elevate the involved area, lities) 50 Footman amentcad -refer you to physiotherapist for strengthening exercises. -Lifestyle modification -if the pain is sever or affected your daily activities--> refer for steroid injection 20. Low lying placenta Your next patient in GP is a 28-year-old pregnant lady, primid with 19 wks of gestation as she wants to discuss the USG finding. USG shows a single baby and a low-lying placenta. 5 cm from the os. «No BPVnor discharge © No abd pain * No medical problem * ANC regular and no prob © Baby kicking (+) + Bld test done + Married and well support family ‘* Previous h/o miscarriage, Sx or procedure **Concerned of pain and prolapsed for NSVP and want to do CS Feedback Pros and cons of NSVP and LSCS- NSVD ~ painless labor with epidural anesthesia, Kegel exercise CS—do not prevent prolapse by doing CS ‘Ask history during counselling instead of taking time for asking history earlier. 21. Post-natal.checkup_gestation DM pt A 28-year-old lady who is 6 weeks post-partum, comes to your GP as a routine checkup. Tasks © History ‘+ Explain about further mx as necessary 51 Footman amentcad Mother ~ everything fine; no fever, breast feeding and no prob, bowel and bladder ~fine, no abd distension or tenderness, no loss PV, Epi wound + and heal well, sleep and appetite — good, mood ‘ok, supportive family Baby — fine, no J, feeding well, pee and poo fine, weight gain and thriving well, active baby Delivery —no problem During preg — gestational DM + Contraception — not sexual activity starts yet, no contraception yet, only breast feeding Feedback + ask if any problems or abnormal tests during pregnancy © OGTT—at 6 wk postnatal in case of gestational DM and regular follow-up for life long as gestational DM increases the risk of DM (3 yearly OGTT) and also increases risk in consecutive preg and needs to be monitored closely ‘Refer to the dietician + Lifestyle modification ‘contraception © Regular follow up 22. Pelvic examination PE 62 years old lady come with brownish discharge. get her consent for pelvic Examination and do the examination. Explain possible diagnosis and necessary inv to pt. Gather the appropriate equipment: there are . Gloves + Lubricant + Paper towels * speculum + cstkit ‘on table Wash your hands and don PPE if appropriate. Introduce yourself to the patient including your name and role. Confirm the patient’s name and date of birth, 52 Footman amentcad Explain what the examination will involve using patient-friendly language: “Today | need to carry out a vaginal examination. This will involve me using one hand to feel your tummy and the other hand to place two fingers into your vagina. This will allow me to assess the vagina, womb and ovaries. It shouldn't be painful, but it will feel a little uncomfortable. You can ask me to stop at any point.” Explain the need for a chaperone: Gain consent to proceed with the examination Ask the patient if they have any pain or if they think they may be pregnant before proceeding with the clinical examination, Provide the patient with the opportunity to pass urine before the examination. ‘An abdominal examination should always be performed before moving onto vaginal examination, This may be less thorough than a full abdominal examination, but should at least include inspection and palpation of the abdomen. Vulval inspection Position 1, Don a pair of non-sterile gloves. 2. Position the patient in the modified lithotomy position: “Bring your heels towards your bottom and then let your knees fall to the sides.” - Inspect the vulva for abnormalities: Ulcers: typically associated with genital herpes. Abnormal vaginal discharge: causes include candidiasis, bacterial vaginosis, chlamydia and gonorrhea. Scarring: may relate to previous surgery (e.g. episiotomy) or lichen sclerosis (destructive scarring with associated adhesions). Vaginal atrophy: most commonly occurs in postmenopausal women. White lesions: may be patchy or in a figure of eight distribution around the vulva and anus, associated with lichen sclerosis. Masses: causes include Bartholin’s cyst and vulval malignancy. Varicosities: varicose veins secondary to chronic venous disease or obstruction in the pelvis (e.g. pelvic malignancy). 53 Footman amentcad Female genital mutilation: total or partial removal of the clitoris and/or labia and/or narrowing of the vaginal introitus. 2. Inspect for evidence of vaginal prolapse (a bulge visible protruding from the vagina). Asking the patient to cough as you inspect can exacerbate the lump and help confirm the presence of prolapse. Vaginal examination Warn the patient you are going to examine the vagina and ask if they're still ok for you to do so. If the patient consents to the continuation of the examination: 1. Lubricate the gloved index and middle fingers of your dominant hand. 2. Carefully separate the labia using the thumb and index finger of your non-dominant hand. 3. Gently insert the gloved index and middle finger of your dominant hand into the vagina. 4. Enter the vagina with your palm facing laterally and then rotate 90 degrees so that your palm is facing upwards. Vaginal walls Palpate the walls of the vagina for any irregularities or masses. Cervix Examine the cervix to assess: Position (e.g. anterior or posterior) Consistency (e.g. irregular, smooth) Cervical motion tenderness: involves severe pain on palpation of the cervix and may suggest pelvic inflammatory disease or ectopic pregnancy. Fornixes The fornixes are the superior portions of the vagina, extending into the recesses created by the vaginal portion of the cervix. Gently palpate lateral fornixes for any masses. Uterus Bimanually palpate the uterus: 54 Footman amentcad 1. Place your non-dominant hand 4cm above the pubis symphysis. 2. Place two of your dominant hand's fingers into the posterior fornix. 3. Push upwards with the internal fingers whilst simultaneously palpating the lower abdomen with your non-dominant hand. You should be able to feel the uterus between your hands. You should then assess the various characteristics of the uterus: Size: the uterus should be approximately orange-sized in an average female. Shape: may be distorted by masses such as large fibroids. Position: the uterus may be anteverted or retroverted. Surface characteristics: note if the uterus feels smooth or nodular. Tenderness: may suggest inflammation (e.g. pelvic inflammatory disease, ectopic pregnancy) Ovaries and uterine tubes The term adnexa refers to the area that includes the ovaries and fallopian tubes. Bimanually palpate the adnexa: 1 Position your internal fingers in the left lateral fornix. 2. Position your external hand onto the left iliac fossa. 3. Perform deep palpation of the left iliac fossa whilst moving your internal fingers upwards and laterally (towards the left). 4, Feel for any palpable masses, noting their size and shape (e.g. ovarian cyst, ovarian tumour, fibroid), 5. Repeat adnexal assessment on the right. 6. Withdraw your fingers and inspect the glove for blood or abnormal discharge. 7. Cover the patient with the sheet, explain that the examination is now complete and provide the patient with privacy so they can get dressed. Provide paper towels for the patient to clean themselves. 8. Dispose of the used equipment into a clinical waste bin. Urinalysis: including B-HCG to rule out pregnancy (including ectopic pregnancy). Speculum examination: to visualize the vaginal canal and cervix. Vaginal swabs/endocervical swabs: if there are concerns about infection (bacterial and viral). 55 Footman amentcad Ultrasound abdomen and pelvis: to better visualize any masses palpated and to assess endometrial thickness. Complete abdominal examination: if there are concerns about intraabdominal pathology (e.. appendicitis). 23. lmmunization counselling You are a GP. A mother, Linda, brought daughter for GP due to mild viral infection. Nurse checked immunization an is not up to date. Mother refused immunization. Her daughter's name is Rhea. Task: Counselling about immunization talk to mother to understand the reason and advice © G months * no fever, some cough ‘+ don’t want to give vaccine as just to rely on natural immunity ‘© why can’t we rely on natural immunity? Approach: Greetings! What would you like to know? How much do you know about immunizations? What is immunization —Important part of preventive medicine, major advantage protection against most childhood infections some of those infections are life-threatening Let me explain you what a vaccine is. Our immune system cor ists of special cells and chemicals which fight against infections. 56 Footman amentcad In addition, we can become immune to diseases either naturally by catching and surviving when contact with illness. Another mechanism to build up resistance to an illness is immunization where we to the patients. ‘+ The vaccine composed of weakened dead microbes/organisms which stimulates an artificial type of infection without getting sick from it, stimulating body to produce Ab. ‘+ Some vaccines ~deactivated toxins. ‘The antibodies produced from body can effectively destroy a microbes which could be virus or bugs or fungus. If they enter the body, because of being recognizing them as enemies, the antibodies destroy them. * Vaccines can provide |i © Most vaccines are given in injections except Rota V vaccine. * Sometimes, numbers of vaccines are combined in one injection. /elong immunity or in some cases, a booster dose is needed. * Vaccines are given according to recommendation which you can find in blue book. SE: redness, swelling, fever, but you can give him Panadol & rest. It will settle down. Absolute contraindications —baby with previous serious anaphylaxis for immunization, Acute neurological illness(encephalopathy) within 7 days of pre-vaccination. Vaccination is totally FOC too. No jab no play: ‘* For child care, school immunization is mandatory. © These vaccines can protect your baby as well as community. Q-why can’t I rely on natural immunity developed? * because some are serious and can lead to severe problems, ‘Although they can provide immunity, ‘the advantage of immunization is definitely a much lower risk and ‘* the child produces antibodies to the disease just like natural immunity. ‘© But in both scenarios, the child may require a booster dose because immunity can drop overtime. ns? Will my child not have any infections after vaccina © Unimmunized baby can become very sick from illness. 57 Footman amentcad ‘+ An immunized baby are less likely to cause severe infections even if they are infected. Child having flu now + only a fever of 38.5 and more is considered to delay vaccination She and her husband searched a lot and found that as it can cause brain damage and fits © this is not true. ‘* This may happen with the previous batches which contained mercury but it doesn’t happen anymore with the ones we are using now. MMR can cause autism ‘there is no link between autism and the MMR vaccine ‘Research also indicates there is no difference in the rates of autism between vaccinated and unvaccinated children. ‘+ While autism may seem more common in recent years, this is due to increased diagnosis stemming from greater awareness about the condition, His older 4yrs brother had local reaction and currently stopped vaccination ‘* unless severe anaphylactic reaction, vaccination shouldn't be stopped In women's gathering most of them were talking about the harm of vaccines ‘* benefits are so much outweighed then the drawbacks as per previous discussion Any questions? 24. Osteoporosis You are a GP, your next patient is 67 year old Mary, who has come to you for back pain that she got 2 days ago while she was picking up a heavy shopping bag. The pain is at the lower end of her back and she took Panadol for 2 days and the pain has not settled yet. Her son has advised her to see a doctor as the pain is getting worse. You had ordered an x-ray of the spine. TASKS -Explain the x-ray to the patient Tell her how that happened 58 Footman amentcad -Explain the implications of this condition on her life, with reasons -Tell her what you are going to do for further management ha Differential diagnosis Itrauma 2-secondary deposits 3-multiple myeloma 4-osteoporsis Brief History LLIntroduce yourself, show empathy and ask for severity of pain then offer pain killers. 2-Back pain questions -when did it start? what were you doing when the pain started? Constant or intermittent? 2-Differential diagnosis que: -any history of trauma? any loss of weight or appetite? Any lumps or bumps around your body? -any weakness or numbness? -how is your bowel motions? -how is your waterworks? 3-Osteoporosis causes questions 59 Footman amentcad -can you tell me briefly about your diet. Do you take dairy products? -do you do regular exercise? -do you smoke? -Do you drink alcohol? How much? -How much coffee do you drink? -any past illnesses? -Do you take any medications -when was your last menstrual periods? Any hot flushes, sweating or mood changes? history of osteoporosis in family or siblings Explain the x-ray This is an x-ray of your lower back. This is a side view that you can see. The spine is made of lots of bones called vertebrae, placed one on top of the other, with a space in between. And normally, all these vertebrae should be of equal height but as you can see here, one of the. vertebrae has a reduced height and this vertebrae, has undergone a wedge compression. The bones also are having a glassy appearance, which means that the bone density has become low. Tell her how that happened There are several possibilities for a wedge compression fracture of the vertebrae. Most likely is osteoporosis. Condition It is a condition that affects the bones; in another word bones with holes. It occurs when bones lose minerals such as calcium more quickly than the body can replace them, causing the bones to become weak, fragile, and more likely to break. Cause/ risk factors there are some contributing factors to osteoporosis then explain. (Menopause, lack of CA in diet, vitamin D low, lack of sunlight exposure, Smoking/ alcohol/ coffee/ lack of exercise, PMH, Medications) 60 Footman amentcad Complications Once osteoporosis occurs, a major complication that you can get, is fractures with trivial trauma. IF you sustain a fracture at this age, it might take a long time to heal other possibilities could be a nasty growth to plasma cells of the bone marrow called multiple myeloma or nasty metastasis of the bone secondary to cancer somewhere. However the most likely due to osteoporosis. Immediate management 1-Confirm that she is having osteoporosis -Order blood tests (FBE, BSL, UCE, LFT, TET) -Vitamin D level -DEXA scan 2-First priority is to treat the wedge compression fracture -Refer to the orthopaedic surgeon to have an assessment of the fracture (if you are GP) admit her to be seen by orthopaedic registrar (if you are at ED) -Put her on adequate painkillers like Panadeineor NSAIDs -A specialist might decide to give you a bracing for 6-12 weeks followed by physical therapy. -if your nerves are getting affected, then surgery would be the option. Long-term management “Lifestyle modifications 2-Supplementation of calcium and vitamin D 3-Put on bisphosphonates or denosumab 4-Attention to fall prevention -refer her to a fall prevention clinic. Summary : in this case the patient was having back pain and x-ray had done showed wedge fracture. after explaining the x-ray you should not think of osteoporosis only as a cause because DEXA has not 61 Footman amentcad been done yet so you need to give all DDX then your immediate management is to perform other Ix and refer to orthopedics specialist then talk about long term management as usual. 25. Implantation bleeding You are an HMO in ED and a 27-years-old Pregnant lady comes in complaining of vaginal bleeding. LMP 7 weeks ago, home UPT positive. Task-— History and order inv within 4 min After 4 min You will get Pefe with inv result Mx Positive findings: © 2wet pad not fully soaked, brown colour, no clot, tummy pain * bleeding stopped now. * Lower tummy pain 3-4 scale + blood test done not yet knowing the result Physical Exam v General appearance: N Vital signs: N ‘Systemic exam: N Abdomer Pelvic exam: Inspection: any bleeding/ discharge/ clots/tissues: Speculum: bleeding +ve, OS closed ‘© Per vaginal: uterine not enlarged and tenderness-ve, adnexal mass nil, CMT -ve v vvv lood on cervix Inv: 72, O negative blood group, Antenatal check up pending. 62 Footman amentcad ‘© Transvaginal USS: no sacs visible, 1.5cm Right ovarian cyst suggesting corpus luteal cyst, no fluid in pouch of douglus. D/D of bleeding during Early Pregnancy: 1. Normal period 2. Threatened miscarriage 3. Incomplete miscarriage 4. Complete miscarriage 5. Missed abortion 6. Ectopic pregnancy 7. Molar pregnancy 8. Bleeding disorder 9. Trauma 10. Blood thinners History: ls the patient hemodynamically stable? OBleeding questions: How long have you been bleeding? What is the colorof the bleed? How many pads have you used so far? [sit fully soaked? Are there any clots or tissues? Any vesicles or grapes? Any discharge along with it? Do you know your blood group (key point)? Associated: tummy pain? Fever/nausea/ vomiting? Any trauma/ hits to the tummy? Utave u been bleeding from anywhere else? Any hxof bleeding disorder? Pregnancy questions: 63 Footman amentcad =Period questions: When was your last menstrual period? Is it regular? *Partner: are u sexually active? Are u in a stable relationship? Is your partner supportive? Early signs of pregnancy: have u been planning for pregnancy? do you have any early morning nausea/ vomiting, breast soreness? Ever been pregnant before? Any hxof miscarriage? Qo you smoke, drink alcohol or take recreational drugs? Dany medical or surgical illness? are u on any kind of medication? EXPLAIN: alt might be various reason, Common in normal pregnancy. It can happen due to implantation. Others possibility about other types of mi arriage like Threatened miscarriage. Incorrect dates. Lit might be ectopic pregnancy, as Bheg is high. Normally the product of pregnancy should be in the womb but when itis outside of your womb like in the tubes we call it ectopic pregnancy. Unfortunately this type of pregnancy is not viable. | am really sorry. Danother reason it Missed abortion because fetus died and still in the womb, Or it might be incomplete miscarriage because some part of your pregnancy might be still in your womb or complete miscarriage / Or might be threatened miscarriage. That’s why | need to admit you to the hospital and call my senior The specialist will come and review u. in the mean time | have already started u with fluids and send your blood for necessary investigations Management: If threatened just take adequate rest do repeat USG 1 week later. Monitor bheg, Now this time consider as implantation bleeding as no other symptoms Observe, adequate rest, SNAP approach. Your blood group has reported as negative, we will give you an injection called anti-D. 64 Footman amentcad I know it is a very hard time for you. Do you want me to calll anyone for you? aR Ectopic pregnancy: Ifitis an ectopic This is an emergency situation cause anytime this tube can rupture and you can have life-threatening bleeding In the mean time | will start you on an IV line, take blood for the investigations FBE, ESR, CRP, UEC, B-HCG, blood group and Rh typing, coagulation profile, start an IV fluid, and arrange for a transvaginal ultrasound. ‘Once confirmed, there are two methods of management for ectopic pregnancy: conservative management and surgical management. Conservative: by methotrexate injection Surgical: to remove the ectopic and repair the tube But it will be decided by the specialist A corpus luteum cyst is a type of ovarian cyst often considered as a functional cyst (a small, fluid- filled sac). The corpus luteum itself is a vital, but temporary endocrine structure. It forms from cells in the ovarian follicle wall during ovulation. This type of cyst happens when the corpus luteum continues to grow rather than break down as it typically does. Corpus luteum cysts are considered, "functional" cysts. This means they typically do not cause harm, rarely need medical intervention, and do not impact fertility Also called a corpus luteal cyst, this condition often occurs during pregnancy. It can also appear at other times, primarily during the reproductive years. Typically, the corpus luteum will disintegrate around 11 to 12 days after ovulation if conception does not occur and at between 7 and 9 weeks during pregnancy. However, sometimes, a corpus luteum cyst can develop on the ovary instead. When this happens the corpus luteum fills with blood and keeps growing rather than breaking down at the typical time. Usually, the cyst is benign, painless, and will eventually go away on its own. Symptoms 65 Footman amentcad In many cases, women who have a corpus luteal cyst do not experience any pain or other symptoms. In fact, a corpus luteal cyst will typically resolve on its own after a few menstrual cycles without a woman even knowing it was there. The most common symptom is a slight twinge of one-sided pain or mild tenderness during the menstrual cycle. However, if there is some pain, it can mimic the type of pain associated with an ectopic or tubal pregnancy Investigation: Corpus luteal cysts are typically diagnosed with a transvaginal ultrasound, also called an internal ultrasound. A transvaginal ultrasound allows for a closer view of pelvic structures than a pelvic ultrasound offers. However, these cysts may also be seen using a pelvic ultrasound, which is a non-invasive diagnostic ultrasound tool that's placed on top of the abdominal skin. Treatment A corpus luteum cyst is usually not harmful. The cysts do not typically cause any complications during pregnancy, especially when they're discovered during the first trimester. if the cyst continuing to grow or worsen, surgery may be required to avoid the risk of miscarriage 26. Primary hyperparathyroidism You are a general practitioner. A 57 years old Margaret coming to you. She has hypothyroidism for 6 years which is well controlled. She had chronic hypercalcaemia for years. She came here today for blood test results. Blood investigation results are as follows: TSH -normal Free T4—normal Corrected calcium increased vv y Task : history © ask more Invx from examiner explain possible Dx with reasons 66 Footman amentcad calcium showed high Lump in neck + No menopausal symp General health good Thyroxine for 150 once a day No stone, groom, moan, bone problem vVvVVVY © TSH-normal © Free T4—normal © Ca-normal © Corrected calcium Increased © PTH-increased © Creatine , eGFR—normal ory: Hypothyroid —which drug? What dose? Well controlled? Any symptoms? (not much needed as well controlled mentioned in the stem) Hypercalcemia ~Causes (PARATHORMONE) v P -Primary and tertiary hyperparathyroidism (to check in Invx) A-amyloidosis R—renal failure -any history of kidney disease? ‘A -Addison’s disease (any diseases that need to need to take steroids) T=TB, Toxoplasmosis H "Histoplasmosis (no need to ask) (0-Overdose of vitamin D (any vit D supplements?) R—Raynaud’s associated disease (no need to ask) M-muscle primaries (no need) (O-ossifying metastases N -Nephrocalcinosis(no need) E-endocrine tumors(gastrinoma) vvv VvVVVVY Features of hypercalcemia ~“bones, stones, psychic moans, abdominal groans” 67 Footman amentcad Any abdominal pain, vomiting? Constipation? How is your water work? Stones? Feel thirsty all the time? Tiredness? weight loss? weakness? Features of hyperparathyroidism —usually asymptom: :, shows only features of hypercalcemia Features of malignancy -loss of weight, loss of appetite, bone pain Past medical, past surgical SADMA Investigations (to differentiate between malignancy and primary hyperparathyroidism as well) © Albumin + URE ‘© Parathyroid levels, ‘Alkaline phosphatase Explain Hypercalcemia due to increased hormone called parathyroid. The gland is all 4 in numbers located near thyroid gland. This hormone plays important role in bone metabolism. Most likely due to adenoma (80%) benign tumor of that gland, hyperplasia of all glands (enlargement of all 4 of the glands) and very less likely cancer. ‘Sometimes this increased parathyroid hormone level occurs in combination as other hormonal disorders (multiple endocrine neoplasia MEN-1 and MEN-2a) but | didn’t notice anything in your history. Explain the studies: lunderstand you are diagnosed with hypothyroidism and you are taking medication for that and the results showing that your thyroid hormone level is in normal level. However, the level of your corrected calcium is slightly higher than the normal limit. | can see your previous blood test result which you have done 6-month back is also showing high calcium level in blood. Which indicate you are having chronic hypercalcemia: Thanks for your cooperation. | would like to talk to examiner. Examiner, | want to order FBE, Liver function test including ALP to see any liver problem, Serum parathyroid hormone level to find out parathyroid gland acting normally or not, 24 hr urinary calcium, UEC, vit D level, RFT to see any kidney problem. 68 footing Management: This high calcium level in your blood can be due to many reason. It can be due to thyrotoxicosis, which is increased thyroid hormone level from medication side effect but unlikely as investigations doesn’t show that. Sometimes it can be due to some other medications but you are not taking anything else. Could be due to some cancer or kidney problem but very much unlikely in your case. | think you are having a condition called Primary hyperparathyroidism where an enlargement of one or more of the parathyroid glands which are 4 small glands located in your neck causes ‘overproduction of the hormone, resulting in high levels of calcium in the blood. Now a noncancerous growth adenoma on a gland is the most common cause, hyperplasia of two or more parathyroid glands accounts for most other cases, a cancerous tumor is a rare cause of primary hyperparathyroidism. Low dietery intake can be cause due to negative feedback Don’t worry it’s a manageable condition. We will manage Refer/ reading material (estan) Fence") PEPE) 1° or 3° hyperpara- thyroiaism rALb (Eg from Tone mi nora es *haveloma (rolasma Bone brotein) metastases - Vitamin o excess Sarcoidosis + Soreoidosis Thyrotoxicosis = With HCO, Lithia mifkealkal Syn rome” Fig 14.5 Hypercalcaemia. 27. POST PARTUM CHECK UP (Post partum PE post 6wks) 69 Footman amentcad G1P1 middle aged lady came in for her 6 weeks postpartum check up. Her baby is all well, and she is purely breastfeeding. Sexual act has been started but she complains of discomfort. There is, also note of minimal passing of urine at times. Patient delivered her baby (~4.2kg?) via assisted vaginal delivery with forceps and episiotomy was done. Tasks: Do relevant PE to the patient for 6mins Positive findings © manually measure BP © Thyroid exam (Examiner: normal) * CVS and Respiratory (Examiner: normal) «Pelvic examination ‘* No lesions outside and episiotomy is healing well with no signs of infections ‘some dryness on the vaginal wall. No CMT ‘uterus size to be within the norm. No masses or tenderness. still no menstrual «bleed. No other d/c abd exam - normal, ut is not papable, no tenderness | started from vital signs, like PR and BP -manually measured BP. Examiner skipped CVS and resp Did actual examination on abdomen commented no tenderness, and did not feel uterus Pelvic exam -non remarkable, but some urine leakage on straining (I said | put gauze over vagina and get the patient to strain), no prolapse on this Reading time: I'm thinking of the “B's” of postpartum (Baby, Blues, Breasts, Birth canal, Bladder, Bowel) that | need to assess. Taken note of the delivery complications and present complains of mild stress incontinence and dry vaginal wall which makes sexual act uncomfortable. Need to check how much the episiotomy degree of tear is as well. Upon entry, | looked right away for a mannequin dummy -there was none. | saw the RP fully clothe. | did my usual greetings to the examiner and RP. Hand sanitized and wore gloves and explained PE and gained consent. 70 Footman amentcad (On GA, described accordingly the patient. Then | asked the Examiner for VS, temp was afebrile. That's all that was given. When asking for pulse rate, | was handed a timer. Took pulse for 15secs and did calculations with estimations of PR in a minute. When asked for BP, | was given the BP app. | thought I'd just do the initial spiel (BP cuff needs to be the right size, at least 40%of the patient’s arm diameter and at least 70-80% arm’s length) then wrapped it unto the patient’s arm, snug with one finger tolerating and was about to pump the cuff with my Steth diaphragm on the Brachial artery --I pumped once and when | realized the ‘examiner not stopping me and giving me BP readings, that’s when it kicked in that | need to do the whole thing | stopped and went back to my commentary -"to accurately check for the BP, | will need to get an initial SBP reading by estimating it -pumping the cuff and checking when the radial pulse will diminish. Once noted of the initial SBP reading, | will then place my Steth on top of Brachial Artery and inflate the cuff around 10-20mmHg above my initial reading and slowly deflate until | hear the 1st Korotkoff sounds and the last Korotkoff sounds which gives the SBP and DBP readings’ this time, my time is so short now. went straight to face: eyes for pallor if evident and if there are signs of dehydration. Thyroid exam (Examiner: normal) CVS and Respiratory (Examiner: normal) ‘Abdominal exam: RP automatically lie down and exposed her abdomen. Normal on inspection. | just quickly palpated superficial then deep (all normal). Have commented that as patient just gave birth, 'm assuming that the uterus is still partially abdominal organ as it descends back to the pelvis. Missed bowel sounds and percussion. Pelvic examination: commented for any rash, swelling outside in the vulvar area and if the episiotomy is healing well (Examiner: No lesions outside and episiotomy is healing well with no signs of infections). ‘Speculum exam: (Examiner: some dryness on the vaginal wall. No CMT). Bimanual exam: (Examiner: commented on uterus size to be within the norm. No masses or tenderness. Still no menstrual bleed. No other d/c). Requested for DRE: Normal. | forgot to do Cough impulse in pelvic exam so | requested but time was up. Have not touched 2nd task n

You might also like